Đến nội dung

LangTu Mua Bui nội dung

Có 43 mục bởi LangTu Mua Bui (Tìm giới hạn từ 14-05-2020)



Sắp theo                Sắp xếp  

#598961 Chứng minh phương trình có nghiệm

Đã gửi bởi LangTu Mua Bui on 18-11-2015 - 17:12 trong Giải tích

xét hàm số $f(x)=\frac{ax^{4}}{4}+bx^{2}+2cx$

Ta có $f(0)=0;f(2)=4a+4b+4c=4(a+b+c)=0 $ Theo đl Roll 

$\Rightarrow f(0)=f(2)\Rightarrow  \exists c \in (0;2) f'(c)=0$

$ \Leftrightarrow pt ax^{3}+2bx+2c=0$có nghiệm$ (0;2)$



#603783 CMR tồn tại 3 số $a,b,c$ sao cho $a< b< c;a+c=2b$...

Đã gửi bởi LangTu Mua Bui on 18-12-2015 - 18:14 trong Giải tích

Cho $f(x)$ liên tục trên R ;giả sử tồn tại 2 số $x_{1};x_{2}$ sao cho $f(x_{1}).f(x_{2})< 0$.

CMR tồn tại 3 số $a,b,c$ sao cho $a< b< c;a+c=2b$ và $15f(a)+2f(b)+2014f(c)=0$

----Nguyên bản BK2014---------


xét hàm g(x)=15f(x+d)+2f(x+2d)+2014f(x+3d)$
Do $f(x)$ liên tục nên$ g(x) $cũng liên tục

Do tồn tại $x_{1}$ và $x_{2} $ sao cho$ f(x_{1})f(x_{2})<0 $

$\Rightarrow \exists (a;c) ;(c;b)  sao cho f(x)<0 \forall x \in (a;c) và f(x)>0 \forall x\in (c;b) ;f(c)=0$

$\Rightarrow \alpha ;d $sao cho$ 15f( \alpha +d)+2f( \alpha +2d)+2014f( \alpha +3d)<0 $

Với đk $\alpha_{1}+d>a;\alpha+3d<c$

Tương tự$ \alpha_{2}+d ...$
 
$(\alpha_{1}).g(\alpha_{2})<0 $nên tồn tại $ \gamma \in (\alpha_{1};\alpha_{2}) $ sao cho $g(\alpha)=0$ Thỏa mãn đề bài 

 



#716595 CM: $\left \{ X\left ( \lambda _{1}...

Đã gửi bởi LangTu Mua Bui on 15-10-2018 - 15:39 trong Đại số tuyến tính, Hình học giải tích

Đặt $ u_{1}=(1,2,1),u_{2}=(2,1,0)$

Xét pt  $ \left ( u_{1}+ \lambda _{1}u_{2} \right ) +a\left ( u_{1}+ \lambda _{2}u_{2} \right )=0 (1)$ nếu tồn tại a thì chúng phụ thuộc tt.

Mặt khác $u_{1},u_{2} $là độc lập tuyến tính trong $R^{3} $ nên $(a+1)u_{1}+(\lambda+a\lambda_{2})=0 $ khi và chỉ khi hay pt (1) =0 khi và chỉ khi $ a=-1,\lambda_{1}=lambda_{2} $ trái đk giả thiết nên pt k tồn tại hay 2 vector đó thể đltt.



#598774 \int_{0}^{1}f(x).x^{n}=0$ với mọi...

Đã gửi bởi LangTu Mua Bui on 17-11-2015 - 15:05 trong Giải tích

$ \Rightarrow \int_{0}^{1}C^{k}_{n}x^{n-k}m^{k}f(x)dx =0 $

$ \Rightarrow \left ( \sum_{k=0}^{n}C^{k}_{n}x^{n-k}m^{k}f(x) \right )=0 $

$\Leftrightarrow \int_{0}^{1}(m+x)^{n}f(x)dx \forall n\in N ;m\in R $

Đặt  $m+x=t \Rightarrow \int_{0}^{m+1}x^{n}f(x-m)dx=0 ;n=1 \Rightarrow \int_{0}^{m+1}f(x-m)dx=0 $;

Đặt $ g(m)=\int_{0}^{m+1}f(x-m)dx=0 \forall m \in R \Rightarrow g(m)=c \Rightarrow g'(x)=0 $ $ \Rightarrow f(1)=0 $




#598770 \int_{0}^{1}f(x).x^{n}=0$ với mọi...

Đã gửi bởi LangTu Mua Bui on 17-11-2015 - 13:27 trong Giải tích

Có thêm điều kiện về f(x) như liên tục hay khả vi gì k bạn 




#599240 Tính giá trị biểu thức $\arctan u+\arctan v+\arctan...

Đã gửi bởi LangTu Mua Bui on 20-11-2015 - 16:15 trong Phương trình - Hệ phương trình - Bất phương trình

Gọi $u,v,w$ là ba nghiệm của phương trình $x^3-10x+11=0$. Tính giá trị của biểu thức $$\arctan u+\arctan v+\arctan w$$.

Tạp chí KoMaL, Hungary.

Ta có đẳng thức quen thuộc $x=\tan{\arctan{x}}$

Biển đổi qua Ta có công thức dễ dàng  

$\Rightarrow \arctan{x}+\arctan{y}=\arctan{\tan{\left ( (\arctan{x})+(\arctan{y}) \right )}} =\arctan{\left ( \frac{\tan{(\arctan{x})}+\tan{(\arctan{y})}}{1+\tan{(\arctan{x})}\tan{(\arctan{y})}} \right )}$
 
Theo đl viet bậc cao 
$\Rightarrow \arctan{x}+\arctan{y}+\arctan{z}=\arctan{\frac{x+y+z+xyz}{1+xy+zx+yz}}$
$\left\{\begin{matrix} x+y+z=\frac{-b}{a}=0\\ xy+yz+zx=\frac{c}{a}=-10 \\zyz=\frac{d}{a}=11 \end{matrix}\right.$

$\arctan{x}+\arctan{y}+\arctan{z}=\arctan{\frac{x+y+z+xyz}{1+xy+zx+yz}}=\arctan{-\frac{11}{9}}$

 



#714733 Chứng minh: $det(J(Y,X))= (detA)^{n}.(detB)^{n}$

Đã gửi bởi LangTu Mua Bui on 23-08-2018 - 23:25 trong Đại số tuyến tính, Hình học giải tích

Viết J(X,Y)=$\frac{\partial y_{i}}{\partial x_{i}}$ là không ổn vì dễ nhầm là phép cuộn .Để tránh nhầm lẫn nên viết lại thành $\frac{\partial y_{i}}{\partial x_{j}}$

Ở bài này nên biết đến tích tensor của 2 ma trận thì dễ hiểu hơn  $J=B^{T} \bigotimes  A$

Biến đổi từ đề bài ta được : $\Rightarrow  y_{ij}= \frac{\partial y_{ih}}{\partial x_{mn}}= \frac{\partial \left ( \sum_{j=1}^{n}\sum_{k=1}^{n}a_{ij}x_{jk}b_{kh} \right ) }{\partial x_{mn}}$

$y_{ij} $ Chỉ khác 0 khi mn trùng với jk .Suy ra ma trận Jacobi biến đổi là 1 ma trận $n^{2}\times n^{2}$

   

Đặt $c_{ij}=b_{ij} $ và  viết dạng ma trận khối 

 

                             $J=\begin{bmatrix} c_{11}A &c_{12}A&.. &c_{nn}A \\ c_{21}A & .. &...& .. \\ ... &...& ...\\ c_{n1}A & ..&.. & c_{nn}A \end{bmatrix}$

 

 

Do cách tính định thức là tổng các tính các hoán vị nên khi nhân các phần tử của $c_{ij}$ với A thì không làm thay đổi hoán vị
Gọi F là trường vector các ma trận vuông cấp n là cấp của mỗi khối , R là trường các ma trận vuông $n^{2} $ của ma trận J 
 

Điều này dẫn đến : $det J=det_{F}(det_{R} J) $

 

                                      $  =det_{F}\left (sum _{\pi \in S_{n}}(sgn \pi)c_{1\pi_{i_{1}}}c_{2\pi_{i_{2}}}...c_{n\pi_{i_{n}}}A^{n} \right )$

 

$det_{F}$ là tính định thức với dạng ma trận khối ,$det_{R}$ là tính định thức coi các ma trận thông thường bằng cách coi các  khối như phần tử vô hướng. 

 

 Do $A^{n} $là ma trận vuông cấp n                 $=det_{F}A^{n}(\left (sum _{\pi \in S_{n}}(sgn \pi)c_{1\pi_{i_{1}}}c_{2\pi_{i_{2}}}...c_{n\pi_{i_{n}}}\right )^{n}$

                                                                          $=detA^{n}detC^{n}$

                                                                          $=detA^{n}(detB^{T})^{n}$

                                                   

                                                                  $  \Rightarrow det J=(detA)^{n}(detB)^{n}$




#605591 câu 1 đề thi OLP giải tích 1997

Đã gửi bởi LangTu Mua Bui on 27-12-2015 - 20:29 trong Thảo luận về các kì thi, các kì kiểm tra Toán sinh viên

cho số thực A khác 0.xét hàm số $f(x)$ xác định;liên tục trên [0;$+\infty $] và thỏa mãn $\underset{x\rightarrow +\infty}{lim}f(x)=A$. tính

$\underset{n\rightarrow +\infty}{lim}\int_{0}^{1}f(nx)dx$ 

Đặt $nx=t\Rightarrow  \int_{0}^{1}f(nx)dx=\frac{\int_{0}^{n}f(n)dt}{n}$

$\Rightarrow \lim_{n\to\infty}\int_{0}^{1}f(nx)dx=\lim_{n\to\infty}\frac{\int_{0}^{n}f(n)dt}{n} (Lopitan)=\lim_{n\to\infty}f(n)=A$



#624370 1,Tính tổng chuỗi $\sum_{n=1}^{vocung }arctan(...

Đã gửi bởi LangTu Mua Bui on 02-04-2016 - 22:12 trong Giải tích

Phải là $\arctan{(\frac{2}{n^{2}})} $chứ nhỉ
$\frac{2}{n^{2}}=\frac{n+1-(n-1))}{1+(n-1)(n+1)}  \Rightarrow \arctan{\left ( \frac{(n+1)-(n-1)}{1+(n+1)(n-1)} \right )}=\arctan{(n+1)}-\arctan{(n-1)}$




#599230 $ \arcsin x +\arcsin x\sqrt{15}=\frac...

Đã gửi bởi LangTu Mua Bui on 20-11-2015 - 15:04 trong Phương trình - Hệ phương trình - Bất phương trình

Giải phương trình $$ \arcsin x +\arcsin (x\sqrt{15})=\frac{\pi}{2} $$

Tạp chí Komal, Hungary.

$\arcsin{x}+\arcsin{x\sqrt{15}}=\frac{\pi}{2}$
Đk :x>0 
Xét hàm $y=\arcsin{x}+\arccos{x} ;y'=0$  y là hàm hằng

$ \Rightarrow y=c $ thay giá trị bất kị vào để tìm c $;y(0)=c=\frac{\pi}{2}$


$\Rightarrow  \arcsin x +\arcsin (x\sqrt{15})=\frac{\pi}{2}$$ \Leftrightarrow \arcsin{x\sqrt{15}}=\arccos{x}$
 
 
Ta có$ t=\arcsin{sin{t}}=\arcsin{(\sqrt{1-\cos^{2}{t}})}$
 
$\arccos{x}$Ở đây Ta hiểu$ x=\cos{t} \Rightarrow \arccos{x}=t $

Ta có $\sin^{2}{t}+\cos^{2}{t}=1 \Rightarrow \sin{t}=\sqrt{1-\cos^{2}{t}}$ Ở đây là xét x>0  $\Rightarrow \arccos{x}=\arcsin{\sqrt{1-x^{2}}}$

$\Rightarrow 15x^{2}=1-x^{2} \Rightarrow x=\dfrac{1}{4}$



#714632 $dimC(A) \ge n$

Đã gửi bởi LangTu Mua Bui on 21-08-2018 - 13:35 trong Đại số tuyến tính, Hình học giải tích

C(A) là tập hợp các ma trận nên  mọi ma trận thuộc C(A) đều thuộc không gian chứa các ma trận vuông $ M_{n\times n }$ thuộc 
Chứng minh dimC(A)>n 
Với không gian các ma trận ta biết số chiều là $n^{2} $

Theo định lý số chiều :

Ta có $dim (Im F(X)) +dim(ker F(X)) =n^{2} $ Vì thế  yêu cầu bài toán tương đương với số chiều $ker F(X) >n $

 

Ta sẽ đi chứng minh dim$ Im(F(X)) <n^{2}-n $

 

Xét ánh xạ  F(X)=AX-XA với X và A là các ma trận vuông cấp n 

 

Ánh xạ F là 1 ánh xạ tuyến tính do nó thỏa mãn 2 tính chất 

 

 Thật vậy :$ F(M+N)=A(M+N)-(M+N)A=AM-MA+AN-NA=F(M)+F(N)$
            và $F(kX)=k(AX-XA)=kF(X) $

 

Bằng biển đổi thông thường  ta luôn có F(X) là 1 ma trận có các phần tử trên đường chéo chính đều bằng 0 với mọi ma trận X 
VÌ vậy số chiều của$ Im F(X) $luôn nhỏ hơn $n^{2}-n$
 Yêu cầu bái toán đã được chứng minh .




#599331 $I=\lim_{n\rightarrow +\infty }\frac{...

Đã gửi bởi LangTu Mua Bui on 20-11-2015 - 23:20 trong Dãy số - Giới hạn

 
 We have $\lim_{n\to\infty}\sqrt[n]{a_{n}}=\lim_{n\to\infty}e^{\frac{\ln{a_{n}}}{n}} $

$=e^{\ln{a_{n+1}-\ln{a_{n}}}}(Stole)=e^{\ln{\frac{a_{n+1}}{a_{n}}}}=\lim_{n\to\infty}\frac{a_{n+1}}{a_{n}}$

$\Rightarrow \lim{n\to\infty}\frac{(n+1)^{n+1}.n!}{(n+1).n!.n^{n}}=\lim_{n\to\infty}(1+\frac{1}{n})^{n}=e$



#599311 Cho $k\in \mathbb{N}^*$. Tìm $$L=\lim_{n...

Đã gửi bởi LangTu Mua Bui on 20-11-2015 - 22:03 trong Giải tích

$\int_{0}^{1}\frac{\ln{(1+x^{n+k})}}{\ln{(1+x^{n})}}=\int_{0}^{1-\varepsilon }\frac{\ln{(1+x^{n+k})}}{\ln{(1+x^{n})}}+\int_{1-\varepsilon }^{1}\frac{\ln{(1+x^{n+k})}}{\ln{(1+x^{n})}} =I_{1}+I_{2} $

$I_{2}=\varepsilon \frac{\ln{(1+{c}^{n+k})}}{\ln{(1+{c}^{n})}}=0 I_{1}=\int_{0}^{1-\varepsilon }\frac{x^{n+k}}{x^{n}}=\int_{0}^{1-\varepsilon }x^{k}$

$=\lim_{\varepsilon \to\ 0}\frac{x^{k+1}}{k+1}|^{1-\varepsilon }_{0} =\frac{1}{k+1}$




#714642 $rank(AB) \le rank(B)-1$

Đã gửi bởi LangTu Mua Bui on 21-08-2018 - 22:24 trong Đại số tuyến tính, Hình học giải tích

Gọi f và g là các ánh xạ tưng ứng với 2 ma trận A và B 

Xét đa thức $P(x)=x^{2017} $ và P(A)=0 suy ra đa thức tối thiểu có dạng $x^{k}$  tồn tại trị riêng $\lambda =0  $  tức $kerf  \neq  0 $

 

$ker fg =kerf +kerg mà ker f \geq 1  \Rightarrow ker fg -1\geq ker B  $

 

Sử dụng thêm định lý dim $ker \varphi  =n-dim im \varphi  $ 
 




#598960 ĐỀ THI GIỮA KÌ MÔN GIẢI TÍCH 20151 (ĐHBKHN)

Đã gửi bởi LangTu Mua Bui on 18-11-2015 - 16:59 trong Giải tích

Khai triển taylor tại $x=2015  \Rightarrow P(x)=P(2015)+\sum_{k=1}^{n}\frac{P^{k}(2015)(x-2015)^{k}}{k!}$

$P(x)=1+\sum_{k=1}^{n}(-1^{k})(x-2015)^{k}=1+\sum_{k=1}^{2015}(2015-x)^{k}$

$\Rightarrow P(2014)=1+2015=2016$




#599255 $\lim_{n\to \infty }\dfrac{a_{n+1}}{a_{n}}=r,\R...

Đã gửi bởi LangTu Mua Bui on 20-11-2015 - 18:39 trong Giải tích

$\lim_{n\to\infty}\sqrt[n]{a_{n}}=\lim_{n\to\infty}e^{\frac{\ln{a_{n}}}{n}} =e^{\dfrac{\ln{a_{n+1}-\ln{a_{n}}}}{n+1-n}}(Stole)=e^{\ln{\frac{a_{n+1}}{a_{n}}}}=\lim_{n\to\infty}\frac{a_{n+1}}{a_{n}}$




#599589 $$\sum_{k = 0}^\infty \frac{2^k...

Đã gửi bởi LangTu Mua Bui on 22-11-2015 - 19:15 trong Dãy số - Giới hạn

Đặt $S=\sum_{i=0}^{k}(1+\sqrt{5})^{k-i}(1-\sqrt{5})^i$.

Ta thấy $S$ là tổng của $k+1$ số hạng đầu của 1 cấp số nhân có $u_1=\left ( 1+\sqrt{5} \right )^k$ và $q=\frac{1-\sqrt{5}}{1+\sqrt{5}}$

$\Rightarrow S=\frac{u_1(q^{k+1}-1)}{q-1}=...=\frac{(1+\sqrt{5})^{k+1}-(1-\sqrt{5})^{k+1}}{2\sqrt{5}}$

$\Rightarrow \frac{2^k}{\sum_{i=0}^{k}(1+\sqrt{5})^{k-i}(1-\sqrt{5})^i}=\frac{2^{k+1}.\sqrt{5}}{(1+\sqrt{5})^{k+1}-(1-\sqrt{5})^{k+1}}=\frac{\sqrt{5}}{\left ( \frac{1+\sqrt{5}}{2} \right )^{k+1}-\left ( \frac{1-\sqrt{5}}{2} \right )^{k+1}}=\frac{1}{F_{k+1}}$

trong đó $F_{k+1}$ là số hạng thứ $k+1$ trong dãy Fibonacci.

Vậy tổng cần tính bằng $\sum_{k=0}^{\infty}\frac{1}{F_{k+1}}=\sum_{k=1}^{\infty}\frac{1}{F_k}\approx 3,359885...$

Cái đoạn chứng minh nghịch đạo dãy đó trên mạng sao k thấy bạn ạ,Bạn có thể chứng minh hãy chỉ rõ hơn không 




#598220 Chứng minh Xn= (1+1/2)(1+1/4)...(1/2^n) có giới hạn.

Đã gửi bởi LangTu Mua Bui on 13-11-2015 - 23:07 trong Giải tích

$x_{n}=\prod_{k=1}^{n}(1+\frac{1}{2^{n}})=2.\left ( 1-\frac{1}{2} \right )\prod_{k=1}^{n}\left ( 1+\frac{1}{2^{k}} \right )=2(1-\frac{1}{2})(1+\frac{1}{2})(1+\frac{1}{4})..(1+\frac{1}{2^{n}})=2\left ( 1-\frac{1}{2^{2^{n}}} \right )\Rightarrow  $ Dãy số có giới hạn =2 




#601388 Tuyển tập một số bài toán Olympic SV Giải tích

Đã gửi bởi LangTu Mua Bui on 03-12-2015 - 14:52 trong Thảo luận về các kì thi, các kì kiểm tra Toán sinh viên

 Câu :3

Đặt 
$a_{n}=\int_{0}^{1}f^{n}(x)dx $

$\Rightarrow \lim_{n \to \infty}\sqrt[n]{a_{n}}=\lim_{n \to \infty}e^{\frac{\ln{a_{n}}}{n}}=\lim_{n \to \infty}e^{\ln{\frac{a_{n+1}}{a_{n}}}}=\lim_{n \to \infty}\frac{a_{n+1}}{a_{n}} $

 

Như ta đã biết $ \int_{0}^{1}f^{n}(x)dx=\sum_{i=0}^{i=n}f^{n}(\frac{i}{n})$

 $\Rightarrow \frac{\int_{0}^{1}f^{n+1}(x)dx}{\int_{0}^{1}f^{n}(x)dx}=\lim_{n\to\infty}\frac{\sum_{i=0}^{i=n}f^{n+1}(\frac{i}{n})}{\sum_{i=0}^{i=n}f^{n}(\frac{i}{n})}$

Do hàm f(x) liên tục$[0;1] \Rightarrow $ tồn tại $x_{0} $ sao cho$ f(x_{0})=maxf(x) ;x_{0} \in [0;1]$

Theo quy tắc ngắt bỏ VCL$\Rightarrow \frac{\int_{0}^{1}f^{n+1}(x)dx}{\int_{0}^{1}f^{n}(x)dx}=\lim_{n\to\infty}\frac{\sum_{i=0}^{i=n}f^{n+1}(\frac{i}{n})}{\sum_{i=0}^{i=n}f^{n}(\frac{i}{n})}

=\lim_{n\to\infty}\dfrac{f^{n+1}(x_{0})}{f^{n}(x_{0})}=f(x_{0})=Maxf(x)$




#601411 Tuyển tập một số bài toán Olympic SV Giải tích

Đã gửi bởi LangTu Mua Bui on 03-12-2015 - 16:57 trong Thảo luận về các kì thi, các kì kiểm tra Toán sinh viên

Câu 7 Như dùng đl Stolez nhưng vẫn có điểm gì đó cần lưu ý 
Câu 5 $u(x)\leq 1+\int_{0}^{x}\frac{\varphi '(t)u(t)dt}{\varphi (t)}$

Dễ thấy $ u(0) \leq 1$
 
$\Leftrightarrow u(x)-\varphi (x)  \leq  \int_{0}^{x} \frac{\varphi '(t)u(t)dt}{\varphi (t)}-\int_{0}^{x}\varphi '(t)dt=\int_{0}^{x}\left (\varphi '(t)( \frac{u(t)-\varphi (t)}{\varphi (t)})  \right ) dt $

Do $\varphi (t)$ đồng biến và $\varphi(0)=1 \Rightarrow \varphi (t)\geq 1 \forall t\in [0;\infty) $

$\Rightarrow u(x)-\varphi (x) \leq  \int_{0}^{x}\left (\varphi '(t)(u(x)-1)  \right ) dt=u(x)-\varphi (x)-\int_{0}^{x}\varphi '(x) $

$\Rightarrow \varphi(x)<1-\int_{0}^{x}u'(t)\varphi (t)dt ;\varphi(x)\geq 1 \forall x\in [0;\infty] \Rightarrow  u'(t)<0 $

Xét hàm số $g(x)=u(x)-\varphi (x) $

$g'(x)=u'(x)-\varphi' (x) <0 ;g(0)=u(0)-\varphi (0)<0 \Rightarrow g(x)<0 \forall x\in [0;\infty]$



#598981 Tính tổng S= $\sum\limits_{k=1}^{2006} f(k...

Đã gửi bởi LangTu Mua Bui on 18-11-2015 - 19:53 trong Các dạng toán THPT khác


$f(n+1)=n(-1)^{n+1}-2f(n) \Leftrightarrow (-1)^{n+1}f(n+1)=n+2(-1^{n})f(n)$

$\Leftrightarrow (-1)^{n+1}f(n+1)+(n+1)=2(n+(-1^{n})f(n))+1 $

$u_{n}=(-1)^{n}f(n)+n \Rightarrow u_{n}=A2^{n}+B$

$u_{1}=-f(1)+1=2A+B;u_{2005}=-f(2005)+2005=A2^{2005}+B$
$\Rightarrow \left\{\begin{matrix} -f(1)+1=2A+B \\-f(2005)+2005=A.2^{2005}+B \\ \end{matrix}\right.$

Tìm được A và B $\Rightarrow f(n)=(-1)^{n}\left ( A.2^{n} +B-n \right )$ Từ đây dễ dàng tính được tổng 



#714327 $rank(A)=rank(B)$

Đã gửi bởi LangTu Mua Bui on 13-08-2018 - 18:22 trong Đại số tuyến tính, Hình học giải tích

Đầu tiên ta đi tìm trị riêng của dạng ma trận đặc biệt này 
Giả sử $\lambda ,x $ lần lượt là trị riêng và vector riêng của A  khi đó ta có biểu thức: 

$$ A^{2}x=Ax \Leftrightarrow \lambda^{2} x=\lambda x $$
$$\Rightarrow \lambda=1 ,\lambda=0$$

 

Hai ma trận là động dạng với nhau khi chúng là biểu diễn của cùng tự đẳng cấu $ f: V\rightarrow  V  $ .Chúng khác nhau do đối với cơ sở khác nhau .

Với $\lambda =1$ là toàn bộ không gian Imf,$\lambda=0 $ là toàn bộ không gian $kerf$ 
Mặt khác ta có định lý số chiều $dim(Imf)+dim(kerf )=n $ Vì thế chúng thỏa mãn điều kiện số chiều nên chéo hóa được 

Và ma trận tương ứng là ma trận đường chéo có các phần tử là 0 và 1 thứ tự của chúng khác nhau tương ứng với cách sắp xếp các vector riêng .

 

VÌ thế 2 ma trận A và B đồng dạng nhau khi chúng có cùng hạng. 




#599258 Chứng minh rằng với mọi n nguyên dương ta có $\dfrac{1}{2n-1} +...

Đã gửi bởi LangTu Mua Bui on 20-11-2015 - 18:56 trong Giải tích

 $f(x+1)-f(x)=f'(c) c\in (x;x+1)Roll$

Xét hàm $f(x)=\ln{x+1} \ln{x+1}-\ln{x}=\frac{1}{c} < \frac{1}{x}c\in (c;c+1)$

$ \Rightarrow \sum_{k=n}^{2n}\frac{1}{k}>\sum_{k=n}^{2n}\ln{(k+1)}-\ln{k}=\ln{2n}-\ln{n}=\ln{2}$



#599209 Đề thi chọn Đội tuyển Olympic SV ĐH Mỏ-Địa chất 2010-2011

Đã gửi bởi LangTu Mua Bui on 20-11-2015 - 10:45 trong Giải tích

Câu 1 $I=\int_{-\dfrac{\Pi }{4}}^{\dfrac{\pi}{4}}ln(tanx+\sqrt{tan^2x+e^{sin^2x}})dx$

Đặt $x=-t  \Rightarrow I= \int_{-\dfrac{\Pi }{4}}^{\dfrac{\pi}{4}}ln(\sqrt{tan^2x+e^{sin^2x}}-\tan{x})dx$
 
$2I=\int_{-\dfrac{\Pi }{4}}^{\dfrac{\pi}{4}}ln(\sqrt{tan^2x+e^{sin^2x}}-\tan{x})dx+\int_{-\dfrac{\Pi }${4}}^{\dfrac{\pi}{4}}ln(tanx+\sqrt{tan^2x+e^{sin^2x}})dx$

$\int_{\frac{-\pi}{4}}^{\frac{\pi}{4}}\ln{e^{\sin^{2}x}}dx=\int_{\frac{-\pi}{4}}^{\frac{\pi}{4}}\frac{(1-\cos{2x})}{2}dx$



#599207 Đề thi chọn Đội tuyển Olympic SV ĐH Mỏ-Địa chất 2010-2011

Đã gửi bởi LangTu Mua Bui on 20-11-2015 - 10:25 trong Giải tích

Câu 4

$f(x)-f(y)=\int_{x+2y}^{2x+y}f(t)dt \Rightarrow f(x)=f(0)+\int_{x}^{2x}f(t)dt$ Nên f(x) là hàm khả vi do theo đn$ f(x)=\int f'(x)+C$ 
 

$f(x)-f(y)=\int_{x+2y}^{2x+y}f(t)dt \Rightarrow f'(x)=2f(2x+y)-f(x+2y) $(Đạo hàm  2 vế theo x )

$\Rightarrow 2f'(x+2y)=2f(2x+y)$( Đạo hàm tiếp  theo biến y)

$\Leftrightarrow f'(x+2y)=f'(2x+y)\Leftrightarrow f((x+y)+y)=f(x+(x+y)) x=\beta -(x+y) y=\alpha -(x+y)$

$\Leftrightarrow f'(\alpha )=f'(\beta ) \forall \alpha ;\beta \in R \Rightarrow f'(x)=C \Rightarrow f(x)=ax+b$